what is the ODE that gives rise to the Laplace Transform.












2












$begingroup$


It can be shown that the transform pairs can be obtained from a differential equation with some boundary conditions. See, for example,
Keener's book chapter 7.



For example. The Fourier transform can be obtained by starting with the
ODE:



begin{eqnarray}
-u''(x) - lambda u(x) = 0 quad , quad lim_{x to pm infty} u(x)=0.
end{eqnarray}
with $u(x) in L^2(-infty, infty)$ .This ODE has a Green function $G(x, xi, lambda)$ with the property that



begin{eqnarray}
delta(x-xi) = - frac{1}{2 pi mathrm{i} } int_{C_{infty}}
G(x,xi, lambda) d lambda
end{eqnarray}



where the contour $C_{infty}$ should enclose all the spectrum of the
operator $L=-d^2/dx^2 - lambda$.



Then the $delta(x-xi)$ comes as the function composition of the forward and inverse transforms.



What is the differential equation (ODE) and its boundary conditions such that the integral of its Green function having all its spectrum (a Dirac delta) is a composition of a forward and inverse Laplace transforms?



Thanks.










share|cite|improve this question









$endgroup$

















    2












    $begingroup$


    It can be shown that the transform pairs can be obtained from a differential equation with some boundary conditions. See, for example,
    Keener's book chapter 7.



    For example. The Fourier transform can be obtained by starting with the
    ODE:



    begin{eqnarray}
    -u''(x) - lambda u(x) = 0 quad , quad lim_{x to pm infty} u(x)=0.
    end{eqnarray}
    with $u(x) in L^2(-infty, infty)$ .This ODE has a Green function $G(x, xi, lambda)$ with the property that



    begin{eqnarray}
    delta(x-xi) = - frac{1}{2 pi mathrm{i} } int_{C_{infty}}
    G(x,xi, lambda) d lambda
    end{eqnarray}



    where the contour $C_{infty}$ should enclose all the spectrum of the
    operator $L=-d^2/dx^2 - lambda$.



    Then the $delta(x-xi)$ comes as the function composition of the forward and inverse transforms.



    What is the differential equation (ODE) and its boundary conditions such that the integral of its Green function having all its spectrum (a Dirac delta) is a composition of a forward and inverse Laplace transforms?



    Thanks.










    share|cite|improve this question









    $endgroup$















      2












      2








      2





      $begingroup$


      It can be shown that the transform pairs can be obtained from a differential equation with some boundary conditions. See, for example,
      Keener's book chapter 7.



      For example. The Fourier transform can be obtained by starting with the
      ODE:



      begin{eqnarray}
      -u''(x) - lambda u(x) = 0 quad , quad lim_{x to pm infty} u(x)=0.
      end{eqnarray}
      with $u(x) in L^2(-infty, infty)$ .This ODE has a Green function $G(x, xi, lambda)$ with the property that



      begin{eqnarray}
      delta(x-xi) = - frac{1}{2 pi mathrm{i} } int_{C_{infty}}
      G(x,xi, lambda) d lambda
      end{eqnarray}



      where the contour $C_{infty}$ should enclose all the spectrum of the
      operator $L=-d^2/dx^2 - lambda$.



      Then the $delta(x-xi)$ comes as the function composition of the forward and inverse transforms.



      What is the differential equation (ODE) and its boundary conditions such that the integral of its Green function having all its spectrum (a Dirac delta) is a composition of a forward and inverse Laplace transforms?



      Thanks.










      share|cite|improve this question









      $endgroup$




      It can be shown that the transform pairs can be obtained from a differential equation with some boundary conditions. See, for example,
      Keener's book chapter 7.



      For example. The Fourier transform can be obtained by starting with the
      ODE:



      begin{eqnarray}
      -u''(x) - lambda u(x) = 0 quad , quad lim_{x to pm infty} u(x)=0.
      end{eqnarray}
      with $u(x) in L^2(-infty, infty)$ .This ODE has a Green function $G(x, xi, lambda)$ with the property that



      begin{eqnarray}
      delta(x-xi) = - frac{1}{2 pi mathrm{i} } int_{C_{infty}}
      G(x,xi, lambda) d lambda
      end{eqnarray}



      where the contour $C_{infty}$ should enclose all the spectrum of the
      operator $L=-d^2/dx^2 - lambda$.



      Then the $delta(x-xi)$ comes as the function composition of the forward and inverse transforms.



      What is the differential equation (ODE) and its boundary conditions such that the integral of its Green function having all its spectrum (a Dirac delta) is a composition of a forward and inverse Laplace transforms?



      Thanks.







      complex-analysis functional-analysis ordinary-differential-equations






      share|cite|improve this question













      share|cite|improve this question











      share|cite|improve this question




      share|cite|improve this question










      asked May 22 '16 at 23:19









      Herman JaramilloHerman Jaramillo

      1,3391019




      1,3391019






















          2 Answers
          2






          active

          oldest

          votes


















          2












          $begingroup$

          The Laplace transform comes from considering $Lf=-f'$ and its resolvent
          $$
          -f'(x)-lambda f(x) = g(x),;;; 0 le x < infty.
          $$
          The spectrum of $L$ is in the left half plane.



          The Laplace transform is a function $F$ that is holomorphic on a right half plane, and nice enough that you end up with the Cauchy representation. For $Re s > gamma$,
          begin{align}
          F(s)& = -frac{1}{2pi i}int_{gamma-iinfty}^{gamma+iinfty}frac{F(z)}{z-s}dz \
          & = frac{1}{2pi i}int_{gamma-iinfty}^{gamma+iinfty}F(z)int_{0}^{infty}e^{t(z-s)}dtdz \
          & = int_{0}^{infty}e^{-ts}left(frac{1}{2pi i}int_{gamma-iinfty}^{gamma+iinfty}e^{tz}F(z)dzright)dt.
          end{align}
          This representation is isometric when you deal with the Hardy class of holomorphic functions $F$ on the right half plane for which
          $$
          |F|_{H^2}^2=sup_{u > 0} int_{-infty}^{infty}|F(u+iv)|^2dv < infty.
          $$
          The holomorphic function is recovered from its $L^2$ boundary function, and then you can write the Cauchy kernel $1/(z-s)$ as shown above; the above steps become rigorous. So this unusual pairing comes from the integral representation of the Cauchy kernel, which is a $delta$ function for holomorphic functions:
          $$
          int_{0}^{infty}e^{-t(s-z)}dt = frac{1}{s-z} = delta_{s}(z).
          $$
          The Laplace transform is inverted by the Bromwich integral.






          share|cite|improve this answer











          $endgroup$













          • $begingroup$
            @TrianAndError : Good trial, no error but I am looking for an ODE,
            $endgroup$
            – Herman Jaramillo
            May 23 '16 at 17:41










          • $begingroup$
            @HermanJaramillo : It comes out the spectral resolution for $frac{d}{dt}$ on $[0,infty)$.
            $endgroup$
            – DisintegratingByParts
            May 23 '16 at 17:58










          • $begingroup$
            @HermanJaramillo : The equation is $-f'-lambda f = 0$ on $[0,infty)$.
            $endgroup$
            – DisintegratingByParts
            May 23 '16 at 18:43












          • $begingroup$
            Actually it is a second order differential equation, and the interval is the whole line. Please observe that I first integrate all along the vertical axis (imaginary) and then at the end use the causel $[0,infty)$ domain. Thanks.
            $endgroup$
            – Herman Jaramillo
            May 24 '16 at 0:05










          • $begingroup$
            @HermanJaramillo : The Fourier transform arises from $-if'-lambda f =0$. You can see that it comes from something else, but the Fourier transform is the spectral resolution of the identity for this differential operator. And the same is true on the finite interval $[-pi,pi]$ with periodic endpoint conditions. And the Laplace transform comes from the first order differential operator on $[0,infty)$. This is especially important when you deal with the Laplace transform, because it is something that cannot arise from symmetric operators with selfadjoint extensions. Not possible.
            $endgroup$
            – DisintegratingByParts
            May 24 '16 at 1:14



















          2












          $begingroup$

          We choose $x$ to be along all the imaginary axis. That is,
          or $x in mathrm{i} (-infty, infty)$. Instead of $x$ we want to think of
          $mathrm{i} x$ for $x$ real, and the substitution $s=mathrm{i} x$.



          The operator (ODE) is $Lu=-u''$, with
          boundary conditions $lim_{x to pm infty} u(x) = 0$.
          Then the spectral problem has as a Green function, the solution to the equation



          begin{eqnarray*}
          -G''(s,xi,lambda) - lambda G(s, xi, lambda) =
          delta(s - xi).
          end{eqnarray*}



          The Green function, in terms of $x$ and $y$, is given by
          (please see my notes on Green functions)



          begin{equation}
          G(x,y, lambda) =-
          frac{ mathrm{e}^{-mathrm{i} sqrt{lambda} |x-y|}}
          {2 mathrm{i} sqrt{lambda}} .
          end{equation}

          where $x,y$ are in the positive real numbers, and in terms of $s=mathrm{i}x$,
          and $t=mathrm{i} y$,



          begin{eqnarray*}
          G(x,y, lambda) = -
          frac{ mathrm{e}^{ sqrt{lambda} |s-t|}}
          {2 mathrm{i} sqrt{lambda}} .
          end{eqnarray*}



          We want to compute the right hand side of the following equation:



          begin{eqnarray*}
          delta(s-t) = -frac{1}{2 pi mathrm{i}} int_{C_{infty}}
          G(s,t,lambda) d lambda
          end{eqnarray*}

          where the contour $C_{infty}$ contains all the spectrum (which is located in the imaginary
          line). As it is, this function as has a
          brunch cut at $0$, so we make the change of variables $lambda=mu^2$, $d lambda=
          2 mu d mu$
          , and unfold the integral where now $mu$ runs from $-infty$ to
          $infty$ along the imaginary axis. We have then



          begin{eqnarray*}
          -frac{1}{2 pi mathrm{i}} int_{C_{infty}}
          G(s,t,lambda) d lambda
          &=& -frac{1}{2 pi mathrm{i}} int_{-mathrm{i} infty}^{mathrm{i} infty}
          2 mu frac{mathrm{e}^{ mu |s -t|}}{2 mathrm{i}
          mu} d mu \
          &=&
          frac{1}{2 pi}
          int_{-mathrm{i} infty}^{mathrm{i} infty}
          mathrm{e}^{mu |s -t|} d mu.
          end{eqnarray*}



          We observe that that there is no need for the absolute bars in the exponent (check
          this by assuming $|s|>|t|$ and then $|t|>|s|$) so we write



          begin{eqnarray*}
          delta(s-t) = frac{1}{2 pi} int_{-mathrm{i} infty}^{mathrm{i} infty}
          mathrm{e}^{mu(s-t)} d mu.
          end{eqnarray*}



          At this moment we assume that $u(x)$ is a causal function, or simple
          a regular (no causal function) multiplied by the Heaviside function $H(x)$.
          Let us now multiply both sides of this equation by $u(x)$ and integrate
          over $s$ between $0$ and $infty$. Then



          begin{eqnarray*}
          u(t) = int_0^{infty} ds , u(s) frac{1}{2 pi} int_{-mathrm{i}infty}^
          {mathrm{i} infty}
          mathrm{e}^{mu(s - t)} d mu.
          = frac{1}{2 pi} int_{-mathrm{i} infty}^{mathrm{i} infty} d mu
          left ( int_0^{infty} ds , u(s) mathrm{e}^{mu s} right )
          ; mathrm{e}^{-mu t}
          end{eqnarray*}



          We define the expression inside parenthesis as $U(mu)$, so that we
          have the Laplace transform pair:



          begin{eqnarray*}
          U(mu) &=& int_0^{infty} ds , u(s) mathrm{e}^{mu s} \
          u(t) &=& frac{1}{2 pi} int_{-mathrm{i} infty}^{mathrm{i} infty} d mu
          U(t) mathrm{e}^{-t mu}.
          end{eqnarray*}






          share|cite|improve this answer











          $endgroup$













            Your Answer





            StackExchange.ifUsing("editor", function () {
            return StackExchange.using("mathjaxEditing", function () {
            StackExchange.MarkdownEditor.creationCallbacks.add(function (editor, postfix) {
            StackExchange.mathjaxEditing.prepareWmdForMathJax(editor, postfix, [["$", "$"], ["\\(","\\)"]]);
            });
            });
            }, "mathjax-editing");

            StackExchange.ready(function() {
            var channelOptions = {
            tags: "".split(" "),
            id: "69"
            };
            initTagRenderer("".split(" "), "".split(" "), channelOptions);

            StackExchange.using("externalEditor", function() {
            // Have to fire editor after snippets, if snippets enabled
            if (StackExchange.settings.snippets.snippetsEnabled) {
            StackExchange.using("snippets", function() {
            createEditor();
            });
            }
            else {
            createEditor();
            }
            });

            function createEditor() {
            StackExchange.prepareEditor({
            heartbeatType: 'answer',
            autoActivateHeartbeat: false,
            convertImagesToLinks: true,
            noModals: true,
            showLowRepImageUploadWarning: true,
            reputationToPostImages: 10,
            bindNavPrevention: true,
            postfix: "",
            imageUploader: {
            brandingHtml: "Powered by u003ca class="icon-imgur-white" href="https://imgur.com/"u003eu003c/au003e",
            contentPolicyHtml: "User contributions licensed under u003ca href="https://creativecommons.org/licenses/by-sa/3.0/"u003ecc by-sa 3.0 with attribution requiredu003c/au003e u003ca href="https://stackoverflow.com/legal/content-policy"u003e(content policy)u003c/au003e",
            allowUrls: true
            },
            noCode: true, onDemand: true,
            discardSelector: ".discard-answer"
            ,immediatelyShowMarkdownHelp:true
            });


            }
            });














            draft saved

            draft discarded


















            StackExchange.ready(
            function () {
            StackExchange.openid.initPostLogin('.new-post-login', 'https%3a%2f%2fmath.stackexchange.com%2fquestions%2f1796021%2fwhat-is-the-ode-that-gives-rise-to-the-laplace-transform%23new-answer', 'question_page');
            }
            );

            Post as a guest















            Required, but never shown

























            2 Answers
            2






            active

            oldest

            votes








            2 Answers
            2






            active

            oldest

            votes









            active

            oldest

            votes






            active

            oldest

            votes









            2












            $begingroup$

            The Laplace transform comes from considering $Lf=-f'$ and its resolvent
            $$
            -f'(x)-lambda f(x) = g(x),;;; 0 le x < infty.
            $$
            The spectrum of $L$ is in the left half plane.



            The Laplace transform is a function $F$ that is holomorphic on a right half plane, and nice enough that you end up with the Cauchy representation. For $Re s > gamma$,
            begin{align}
            F(s)& = -frac{1}{2pi i}int_{gamma-iinfty}^{gamma+iinfty}frac{F(z)}{z-s}dz \
            & = frac{1}{2pi i}int_{gamma-iinfty}^{gamma+iinfty}F(z)int_{0}^{infty}e^{t(z-s)}dtdz \
            & = int_{0}^{infty}e^{-ts}left(frac{1}{2pi i}int_{gamma-iinfty}^{gamma+iinfty}e^{tz}F(z)dzright)dt.
            end{align}
            This representation is isometric when you deal with the Hardy class of holomorphic functions $F$ on the right half plane for which
            $$
            |F|_{H^2}^2=sup_{u > 0} int_{-infty}^{infty}|F(u+iv)|^2dv < infty.
            $$
            The holomorphic function is recovered from its $L^2$ boundary function, and then you can write the Cauchy kernel $1/(z-s)$ as shown above; the above steps become rigorous. So this unusual pairing comes from the integral representation of the Cauchy kernel, which is a $delta$ function for holomorphic functions:
            $$
            int_{0}^{infty}e^{-t(s-z)}dt = frac{1}{s-z} = delta_{s}(z).
            $$
            The Laplace transform is inverted by the Bromwich integral.






            share|cite|improve this answer











            $endgroup$













            • $begingroup$
              @TrianAndError : Good trial, no error but I am looking for an ODE,
              $endgroup$
              – Herman Jaramillo
              May 23 '16 at 17:41










            • $begingroup$
              @HermanJaramillo : It comes out the spectral resolution for $frac{d}{dt}$ on $[0,infty)$.
              $endgroup$
              – DisintegratingByParts
              May 23 '16 at 17:58










            • $begingroup$
              @HermanJaramillo : The equation is $-f'-lambda f = 0$ on $[0,infty)$.
              $endgroup$
              – DisintegratingByParts
              May 23 '16 at 18:43












            • $begingroup$
              Actually it is a second order differential equation, and the interval is the whole line. Please observe that I first integrate all along the vertical axis (imaginary) and then at the end use the causel $[0,infty)$ domain. Thanks.
              $endgroup$
              – Herman Jaramillo
              May 24 '16 at 0:05










            • $begingroup$
              @HermanJaramillo : The Fourier transform arises from $-if'-lambda f =0$. You can see that it comes from something else, but the Fourier transform is the spectral resolution of the identity for this differential operator. And the same is true on the finite interval $[-pi,pi]$ with periodic endpoint conditions. And the Laplace transform comes from the first order differential operator on $[0,infty)$. This is especially important when you deal with the Laplace transform, because it is something that cannot arise from symmetric operators with selfadjoint extensions. Not possible.
              $endgroup$
              – DisintegratingByParts
              May 24 '16 at 1:14
















            2












            $begingroup$

            The Laplace transform comes from considering $Lf=-f'$ and its resolvent
            $$
            -f'(x)-lambda f(x) = g(x),;;; 0 le x < infty.
            $$
            The spectrum of $L$ is in the left half plane.



            The Laplace transform is a function $F$ that is holomorphic on a right half plane, and nice enough that you end up with the Cauchy representation. For $Re s > gamma$,
            begin{align}
            F(s)& = -frac{1}{2pi i}int_{gamma-iinfty}^{gamma+iinfty}frac{F(z)}{z-s}dz \
            & = frac{1}{2pi i}int_{gamma-iinfty}^{gamma+iinfty}F(z)int_{0}^{infty}e^{t(z-s)}dtdz \
            & = int_{0}^{infty}e^{-ts}left(frac{1}{2pi i}int_{gamma-iinfty}^{gamma+iinfty}e^{tz}F(z)dzright)dt.
            end{align}
            This representation is isometric when you deal with the Hardy class of holomorphic functions $F$ on the right half plane for which
            $$
            |F|_{H^2}^2=sup_{u > 0} int_{-infty}^{infty}|F(u+iv)|^2dv < infty.
            $$
            The holomorphic function is recovered from its $L^2$ boundary function, and then you can write the Cauchy kernel $1/(z-s)$ as shown above; the above steps become rigorous. So this unusual pairing comes from the integral representation of the Cauchy kernel, which is a $delta$ function for holomorphic functions:
            $$
            int_{0}^{infty}e^{-t(s-z)}dt = frac{1}{s-z} = delta_{s}(z).
            $$
            The Laplace transform is inverted by the Bromwich integral.






            share|cite|improve this answer











            $endgroup$













            • $begingroup$
              @TrianAndError : Good trial, no error but I am looking for an ODE,
              $endgroup$
              – Herman Jaramillo
              May 23 '16 at 17:41










            • $begingroup$
              @HermanJaramillo : It comes out the spectral resolution for $frac{d}{dt}$ on $[0,infty)$.
              $endgroup$
              – DisintegratingByParts
              May 23 '16 at 17:58










            • $begingroup$
              @HermanJaramillo : The equation is $-f'-lambda f = 0$ on $[0,infty)$.
              $endgroup$
              – DisintegratingByParts
              May 23 '16 at 18:43












            • $begingroup$
              Actually it is a second order differential equation, and the interval is the whole line. Please observe that I first integrate all along the vertical axis (imaginary) and then at the end use the causel $[0,infty)$ domain. Thanks.
              $endgroup$
              – Herman Jaramillo
              May 24 '16 at 0:05










            • $begingroup$
              @HermanJaramillo : The Fourier transform arises from $-if'-lambda f =0$. You can see that it comes from something else, but the Fourier transform is the spectral resolution of the identity for this differential operator. And the same is true on the finite interval $[-pi,pi]$ with periodic endpoint conditions. And the Laplace transform comes from the first order differential operator on $[0,infty)$. This is especially important when you deal with the Laplace transform, because it is something that cannot arise from symmetric operators with selfadjoint extensions. Not possible.
              $endgroup$
              – DisintegratingByParts
              May 24 '16 at 1:14














            2












            2








            2





            $begingroup$

            The Laplace transform comes from considering $Lf=-f'$ and its resolvent
            $$
            -f'(x)-lambda f(x) = g(x),;;; 0 le x < infty.
            $$
            The spectrum of $L$ is in the left half plane.



            The Laplace transform is a function $F$ that is holomorphic on a right half plane, and nice enough that you end up with the Cauchy representation. For $Re s > gamma$,
            begin{align}
            F(s)& = -frac{1}{2pi i}int_{gamma-iinfty}^{gamma+iinfty}frac{F(z)}{z-s}dz \
            & = frac{1}{2pi i}int_{gamma-iinfty}^{gamma+iinfty}F(z)int_{0}^{infty}e^{t(z-s)}dtdz \
            & = int_{0}^{infty}e^{-ts}left(frac{1}{2pi i}int_{gamma-iinfty}^{gamma+iinfty}e^{tz}F(z)dzright)dt.
            end{align}
            This representation is isometric when you deal with the Hardy class of holomorphic functions $F$ on the right half plane for which
            $$
            |F|_{H^2}^2=sup_{u > 0} int_{-infty}^{infty}|F(u+iv)|^2dv < infty.
            $$
            The holomorphic function is recovered from its $L^2$ boundary function, and then you can write the Cauchy kernel $1/(z-s)$ as shown above; the above steps become rigorous. So this unusual pairing comes from the integral representation of the Cauchy kernel, which is a $delta$ function for holomorphic functions:
            $$
            int_{0}^{infty}e^{-t(s-z)}dt = frac{1}{s-z} = delta_{s}(z).
            $$
            The Laplace transform is inverted by the Bromwich integral.






            share|cite|improve this answer











            $endgroup$



            The Laplace transform comes from considering $Lf=-f'$ and its resolvent
            $$
            -f'(x)-lambda f(x) = g(x),;;; 0 le x < infty.
            $$
            The spectrum of $L$ is in the left half plane.



            The Laplace transform is a function $F$ that is holomorphic on a right half plane, and nice enough that you end up with the Cauchy representation. For $Re s > gamma$,
            begin{align}
            F(s)& = -frac{1}{2pi i}int_{gamma-iinfty}^{gamma+iinfty}frac{F(z)}{z-s}dz \
            & = frac{1}{2pi i}int_{gamma-iinfty}^{gamma+iinfty}F(z)int_{0}^{infty}e^{t(z-s)}dtdz \
            & = int_{0}^{infty}e^{-ts}left(frac{1}{2pi i}int_{gamma-iinfty}^{gamma+iinfty}e^{tz}F(z)dzright)dt.
            end{align}
            This representation is isometric when you deal with the Hardy class of holomorphic functions $F$ on the right half plane for which
            $$
            |F|_{H^2}^2=sup_{u > 0} int_{-infty}^{infty}|F(u+iv)|^2dv < infty.
            $$
            The holomorphic function is recovered from its $L^2$ boundary function, and then you can write the Cauchy kernel $1/(z-s)$ as shown above; the above steps become rigorous. So this unusual pairing comes from the integral representation of the Cauchy kernel, which is a $delta$ function for holomorphic functions:
            $$
            int_{0}^{infty}e^{-t(s-z)}dt = frac{1}{s-z} = delta_{s}(z).
            $$
            The Laplace transform is inverted by the Bromwich integral.







            share|cite|improve this answer














            share|cite|improve this answer



            share|cite|improve this answer








            edited Apr 24 '18 at 20:01

























            answered May 23 '16 at 17:04









            DisintegratingByPartsDisintegratingByParts

            59.7k42581




            59.7k42581












            • $begingroup$
              @TrianAndError : Good trial, no error but I am looking for an ODE,
              $endgroup$
              – Herman Jaramillo
              May 23 '16 at 17:41










            • $begingroup$
              @HermanJaramillo : It comes out the spectral resolution for $frac{d}{dt}$ on $[0,infty)$.
              $endgroup$
              – DisintegratingByParts
              May 23 '16 at 17:58










            • $begingroup$
              @HermanJaramillo : The equation is $-f'-lambda f = 0$ on $[0,infty)$.
              $endgroup$
              – DisintegratingByParts
              May 23 '16 at 18:43












            • $begingroup$
              Actually it is a second order differential equation, and the interval is the whole line. Please observe that I first integrate all along the vertical axis (imaginary) and then at the end use the causel $[0,infty)$ domain. Thanks.
              $endgroup$
              – Herman Jaramillo
              May 24 '16 at 0:05










            • $begingroup$
              @HermanJaramillo : The Fourier transform arises from $-if'-lambda f =0$. You can see that it comes from something else, but the Fourier transform is the spectral resolution of the identity for this differential operator. And the same is true on the finite interval $[-pi,pi]$ with periodic endpoint conditions. And the Laplace transform comes from the first order differential operator on $[0,infty)$. This is especially important when you deal with the Laplace transform, because it is something that cannot arise from symmetric operators with selfadjoint extensions. Not possible.
              $endgroup$
              – DisintegratingByParts
              May 24 '16 at 1:14


















            • $begingroup$
              @TrianAndError : Good trial, no error but I am looking for an ODE,
              $endgroup$
              – Herman Jaramillo
              May 23 '16 at 17:41










            • $begingroup$
              @HermanJaramillo : It comes out the spectral resolution for $frac{d}{dt}$ on $[0,infty)$.
              $endgroup$
              – DisintegratingByParts
              May 23 '16 at 17:58










            • $begingroup$
              @HermanJaramillo : The equation is $-f'-lambda f = 0$ on $[0,infty)$.
              $endgroup$
              – DisintegratingByParts
              May 23 '16 at 18:43












            • $begingroup$
              Actually it is a second order differential equation, and the interval is the whole line. Please observe that I first integrate all along the vertical axis (imaginary) and then at the end use the causel $[0,infty)$ domain. Thanks.
              $endgroup$
              – Herman Jaramillo
              May 24 '16 at 0:05










            • $begingroup$
              @HermanJaramillo : The Fourier transform arises from $-if'-lambda f =0$. You can see that it comes from something else, but the Fourier transform is the spectral resolution of the identity for this differential operator. And the same is true on the finite interval $[-pi,pi]$ with periodic endpoint conditions. And the Laplace transform comes from the first order differential operator on $[0,infty)$. This is especially important when you deal with the Laplace transform, because it is something that cannot arise from symmetric operators with selfadjoint extensions. Not possible.
              $endgroup$
              – DisintegratingByParts
              May 24 '16 at 1:14
















            $begingroup$
            @TrianAndError : Good trial, no error but I am looking for an ODE,
            $endgroup$
            – Herman Jaramillo
            May 23 '16 at 17:41




            $begingroup$
            @TrianAndError : Good trial, no error but I am looking for an ODE,
            $endgroup$
            – Herman Jaramillo
            May 23 '16 at 17:41












            $begingroup$
            @HermanJaramillo : It comes out the spectral resolution for $frac{d}{dt}$ on $[0,infty)$.
            $endgroup$
            – DisintegratingByParts
            May 23 '16 at 17:58




            $begingroup$
            @HermanJaramillo : It comes out the spectral resolution for $frac{d}{dt}$ on $[0,infty)$.
            $endgroup$
            – DisintegratingByParts
            May 23 '16 at 17:58












            $begingroup$
            @HermanJaramillo : The equation is $-f'-lambda f = 0$ on $[0,infty)$.
            $endgroup$
            – DisintegratingByParts
            May 23 '16 at 18:43






            $begingroup$
            @HermanJaramillo : The equation is $-f'-lambda f = 0$ on $[0,infty)$.
            $endgroup$
            – DisintegratingByParts
            May 23 '16 at 18:43














            $begingroup$
            Actually it is a second order differential equation, and the interval is the whole line. Please observe that I first integrate all along the vertical axis (imaginary) and then at the end use the causel $[0,infty)$ domain. Thanks.
            $endgroup$
            – Herman Jaramillo
            May 24 '16 at 0:05




            $begingroup$
            Actually it is a second order differential equation, and the interval is the whole line. Please observe that I first integrate all along the vertical axis (imaginary) and then at the end use the causel $[0,infty)$ domain. Thanks.
            $endgroup$
            – Herman Jaramillo
            May 24 '16 at 0:05












            $begingroup$
            @HermanJaramillo : The Fourier transform arises from $-if'-lambda f =0$. You can see that it comes from something else, but the Fourier transform is the spectral resolution of the identity for this differential operator. And the same is true on the finite interval $[-pi,pi]$ with periodic endpoint conditions. And the Laplace transform comes from the first order differential operator on $[0,infty)$. This is especially important when you deal with the Laplace transform, because it is something that cannot arise from symmetric operators with selfadjoint extensions. Not possible.
            $endgroup$
            – DisintegratingByParts
            May 24 '16 at 1:14




            $begingroup$
            @HermanJaramillo : The Fourier transform arises from $-if'-lambda f =0$. You can see that it comes from something else, but the Fourier transform is the spectral resolution of the identity for this differential operator. And the same is true on the finite interval $[-pi,pi]$ with periodic endpoint conditions. And the Laplace transform comes from the first order differential operator on $[0,infty)$. This is especially important when you deal with the Laplace transform, because it is something that cannot arise from symmetric operators with selfadjoint extensions. Not possible.
            $endgroup$
            – DisintegratingByParts
            May 24 '16 at 1:14











            2












            $begingroup$

            We choose $x$ to be along all the imaginary axis. That is,
            or $x in mathrm{i} (-infty, infty)$. Instead of $x$ we want to think of
            $mathrm{i} x$ for $x$ real, and the substitution $s=mathrm{i} x$.



            The operator (ODE) is $Lu=-u''$, with
            boundary conditions $lim_{x to pm infty} u(x) = 0$.
            Then the spectral problem has as a Green function, the solution to the equation



            begin{eqnarray*}
            -G''(s,xi,lambda) - lambda G(s, xi, lambda) =
            delta(s - xi).
            end{eqnarray*}



            The Green function, in terms of $x$ and $y$, is given by
            (please see my notes on Green functions)



            begin{equation}
            G(x,y, lambda) =-
            frac{ mathrm{e}^{-mathrm{i} sqrt{lambda} |x-y|}}
            {2 mathrm{i} sqrt{lambda}} .
            end{equation}

            where $x,y$ are in the positive real numbers, and in terms of $s=mathrm{i}x$,
            and $t=mathrm{i} y$,



            begin{eqnarray*}
            G(x,y, lambda) = -
            frac{ mathrm{e}^{ sqrt{lambda} |s-t|}}
            {2 mathrm{i} sqrt{lambda}} .
            end{eqnarray*}



            We want to compute the right hand side of the following equation:



            begin{eqnarray*}
            delta(s-t) = -frac{1}{2 pi mathrm{i}} int_{C_{infty}}
            G(s,t,lambda) d lambda
            end{eqnarray*}

            where the contour $C_{infty}$ contains all the spectrum (which is located in the imaginary
            line). As it is, this function as has a
            brunch cut at $0$, so we make the change of variables $lambda=mu^2$, $d lambda=
            2 mu d mu$
            , and unfold the integral where now $mu$ runs from $-infty$ to
            $infty$ along the imaginary axis. We have then



            begin{eqnarray*}
            -frac{1}{2 pi mathrm{i}} int_{C_{infty}}
            G(s,t,lambda) d lambda
            &=& -frac{1}{2 pi mathrm{i}} int_{-mathrm{i} infty}^{mathrm{i} infty}
            2 mu frac{mathrm{e}^{ mu |s -t|}}{2 mathrm{i}
            mu} d mu \
            &=&
            frac{1}{2 pi}
            int_{-mathrm{i} infty}^{mathrm{i} infty}
            mathrm{e}^{mu |s -t|} d mu.
            end{eqnarray*}



            We observe that that there is no need for the absolute bars in the exponent (check
            this by assuming $|s|>|t|$ and then $|t|>|s|$) so we write



            begin{eqnarray*}
            delta(s-t) = frac{1}{2 pi} int_{-mathrm{i} infty}^{mathrm{i} infty}
            mathrm{e}^{mu(s-t)} d mu.
            end{eqnarray*}



            At this moment we assume that $u(x)$ is a causal function, or simple
            a regular (no causal function) multiplied by the Heaviside function $H(x)$.
            Let us now multiply both sides of this equation by $u(x)$ and integrate
            over $s$ between $0$ and $infty$. Then



            begin{eqnarray*}
            u(t) = int_0^{infty} ds , u(s) frac{1}{2 pi} int_{-mathrm{i}infty}^
            {mathrm{i} infty}
            mathrm{e}^{mu(s - t)} d mu.
            = frac{1}{2 pi} int_{-mathrm{i} infty}^{mathrm{i} infty} d mu
            left ( int_0^{infty} ds , u(s) mathrm{e}^{mu s} right )
            ; mathrm{e}^{-mu t}
            end{eqnarray*}



            We define the expression inside parenthesis as $U(mu)$, so that we
            have the Laplace transform pair:



            begin{eqnarray*}
            U(mu) &=& int_0^{infty} ds , u(s) mathrm{e}^{mu s} \
            u(t) &=& frac{1}{2 pi} int_{-mathrm{i} infty}^{mathrm{i} infty} d mu
            U(t) mathrm{e}^{-t mu}.
            end{eqnarray*}






            share|cite|improve this answer











            $endgroup$


















              2












              $begingroup$

              We choose $x$ to be along all the imaginary axis. That is,
              or $x in mathrm{i} (-infty, infty)$. Instead of $x$ we want to think of
              $mathrm{i} x$ for $x$ real, and the substitution $s=mathrm{i} x$.



              The operator (ODE) is $Lu=-u''$, with
              boundary conditions $lim_{x to pm infty} u(x) = 0$.
              Then the spectral problem has as a Green function, the solution to the equation



              begin{eqnarray*}
              -G''(s,xi,lambda) - lambda G(s, xi, lambda) =
              delta(s - xi).
              end{eqnarray*}



              The Green function, in terms of $x$ and $y$, is given by
              (please see my notes on Green functions)



              begin{equation}
              G(x,y, lambda) =-
              frac{ mathrm{e}^{-mathrm{i} sqrt{lambda} |x-y|}}
              {2 mathrm{i} sqrt{lambda}} .
              end{equation}

              where $x,y$ are in the positive real numbers, and in terms of $s=mathrm{i}x$,
              and $t=mathrm{i} y$,



              begin{eqnarray*}
              G(x,y, lambda) = -
              frac{ mathrm{e}^{ sqrt{lambda} |s-t|}}
              {2 mathrm{i} sqrt{lambda}} .
              end{eqnarray*}



              We want to compute the right hand side of the following equation:



              begin{eqnarray*}
              delta(s-t) = -frac{1}{2 pi mathrm{i}} int_{C_{infty}}
              G(s,t,lambda) d lambda
              end{eqnarray*}

              where the contour $C_{infty}$ contains all the spectrum (which is located in the imaginary
              line). As it is, this function as has a
              brunch cut at $0$, so we make the change of variables $lambda=mu^2$, $d lambda=
              2 mu d mu$
              , and unfold the integral where now $mu$ runs from $-infty$ to
              $infty$ along the imaginary axis. We have then



              begin{eqnarray*}
              -frac{1}{2 pi mathrm{i}} int_{C_{infty}}
              G(s,t,lambda) d lambda
              &=& -frac{1}{2 pi mathrm{i}} int_{-mathrm{i} infty}^{mathrm{i} infty}
              2 mu frac{mathrm{e}^{ mu |s -t|}}{2 mathrm{i}
              mu} d mu \
              &=&
              frac{1}{2 pi}
              int_{-mathrm{i} infty}^{mathrm{i} infty}
              mathrm{e}^{mu |s -t|} d mu.
              end{eqnarray*}



              We observe that that there is no need for the absolute bars in the exponent (check
              this by assuming $|s|>|t|$ and then $|t|>|s|$) so we write



              begin{eqnarray*}
              delta(s-t) = frac{1}{2 pi} int_{-mathrm{i} infty}^{mathrm{i} infty}
              mathrm{e}^{mu(s-t)} d mu.
              end{eqnarray*}



              At this moment we assume that $u(x)$ is a causal function, or simple
              a regular (no causal function) multiplied by the Heaviside function $H(x)$.
              Let us now multiply both sides of this equation by $u(x)$ and integrate
              over $s$ between $0$ and $infty$. Then



              begin{eqnarray*}
              u(t) = int_0^{infty} ds , u(s) frac{1}{2 pi} int_{-mathrm{i}infty}^
              {mathrm{i} infty}
              mathrm{e}^{mu(s - t)} d mu.
              = frac{1}{2 pi} int_{-mathrm{i} infty}^{mathrm{i} infty} d mu
              left ( int_0^{infty} ds , u(s) mathrm{e}^{mu s} right )
              ; mathrm{e}^{-mu t}
              end{eqnarray*}



              We define the expression inside parenthesis as $U(mu)$, so that we
              have the Laplace transform pair:



              begin{eqnarray*}
              U(mu) &=& int_0^{infty} ds , u(s) mathrm{e}^{mu s} \
              u(t) &=& frac{1}{2 pi} int_{-mathrm{i} infty}^{mathrm{i} infty} d mu
              U(t) mathrm{e}^{-t mu}.
              end{eqnarray*}






              share|cite|improve this answer











              $endgroup$
















                2












                2








                2





                $begingroup$

                We choose $x$ to be along all the imaginary axis. That is,
                or $x in mathrm{i} (-infty, infty)$. Instead of $x$ we want to think of
                $mathrm{i} x$ for $x$ real, and the substitution $s=mathrm{i} x$.



                The operator (ODE) is $Lu=-u''$, with
                boundary conditions $lim_{x to pm infty} u(x) = 0$.
                Then the spectral problem has as a Green function, the solution to the equation



                begin{eqnarray*}
                -G''(s,xi,lambda) - lambda G(s, xi, lambda) =
                delta(s - xi).
                end{eqnarray*}



                The Green function, in terms of $x$ and $y$, is given by
                (please see my notes on Green functions)



                begin{equation}
                G(x,y, lambda) =-
                frac{ mathrm{e}^{-mathrm{i} sqrt{lambda} |x-y|}}
                {2 mathrm{i} sqrt{lambda}} .
                end{equation}

                where $x,y$ are in the positive real numbers, and in terms of $s=mathrm{i}x$,
                and $t=mathrm{i} y$,



                begin{eqnarray*}
                G(x,y, lambda) = -
                frac{ mathrm{e}^{ sqrt{lambda} |s-t|}}
                {2 mathrm{i} sqrt{lambda}} .
                end{eqnarray*}



                We want to compute the right hand side of the following equation:



                begin{eqnarray*}
                delta(s-t) = -frac{1}{2 pi mathrm{i}} int_{C_{infty}}
                G(s,t,lambda) d lambda
                end{eqnarray*}

                where the contour $C_{infty}$ contains all the spectrum (which is located in the imaginary
                line). As it is, this function as has a
                brunch cut at $0$, so we make the change of variables $lambda=mu^2$, $d lambda=
                2 mu d mu$
                , and unfold the integral where now $mu$ runs from $-infty$ to
                $infty$ along the imaginary axis. We have then



                begin{eqnarray*}
                -frac{1}{2 pi mathrm{i}} int_{C_{infty}}
                G(s,t,lambda) d lambda
                &=& -frac{1}{2 pi mathrm{i}} int_{-mathrm{i} infty}^{mathrm{i} infty}
                2 mu frac{mathrm{e}^{ mu |s -t|}}{2 mathrm{i}
                mu} d mu \
                &=&
                frac{1}{2 pi}
                int_{-mathrm{i} infty}^{mathrm{i} infty}
                mathrm{e}^{mu |s -t|} d mu.
                end{eqnarray*}



                We observe that that there is no need for the absolute bars in the exponent (check
                this by assuming $|s|>|t|$ and then $|t|>|s|$) so we write



                begin{eqnarray*}
                delta(s-t) = frac{1}{2 pi} int_{-mathrm{i} infty}^{mathrm{i} infty}
                mathrm{e}^{mu(s-t)} d mu.
                end{eqnarray*}



                At this moment we assume that $u(x)$ is a causal function, or simple
                a regular (no causal function) multiplied by the Heaviside function $H(x)$.
                Let us now multiply both sides of this equation by $u(x)$ and integrate
                over $s$ between $0$ and $infty$. Then



                begin{eqnarray*}
                u(t) = int_0^{infty} ds , u(s) frac{1}{2 pi} int_{-mathrm{i}infty}^
                {mathrm{i} infty}
                mathrm{e}^{mu(s - t)} d mu.
                = frac{1}{2 pi} int_{-mathrm{i} infty}^{mathrm{i} infty} d mu
                left ( int_0^{infty} ds , u(s) mathrm{e}^{mu s} right )
                ; mathrm{e}^{-mu t}
                end{eqnarray*}



                We define the expression inside parenthesis as $U(mu)$, so that we
                have the Laplace transform pair:



                begin{eqnarray*}
                U(mu) &=& int_0^{infty} ds , u(s) mathrm{e}^{mu s} \
                u(t) &=& frac{1}{2 pi} int_{-mathrm{i} infty}^{mathrm{i} infty} d mu
                U(t) mathrm{e}^{-t mu}.
                end{eqnarray*}






                share|cite|improve this answer











                $endgroup$



                We choose $x$ to be along all the imaginary axis. That is,
                or $x in mathrm{i} (-infty, infty)$. Instead of $x$ we want to think of
                $mathrm{i} x$ for $x$ real, and the substitution $s=mathrm{i} x$.



                The operator (ODE) is $Lu=-u''$, with
                boundary conditions $lim_{x to pm infty} u(x) = 0$.
                Then the spectral problem has as a Green function, the solution to the equation



                begin{eqnarray*}
                -G''(s,xi,lambda) - lambda G(s, xi, lambda) =
                delta(s - xi).
                end{eqnarray*}



                The Green function, in terms of $x$ and $y$, is given by
                (please see my notes on Green functions)



                begin{equation}
                G(x,y, lambda) =-
                frac{ mathrm{e}^{-mathrm{i} sqrt{lambda} |x-y|}}
                {2 mathrm{i} sqrt{lambda}} .
                end{equation}

                where $x,y$ are in the positive real numbers, and in terms of $s=mathrm{i}x$,
                and $t=mathrm{i} y$,



                begin{eqnarray*}
                G(x,y, lambda) = -
                frac{ mathrm{e}^{ sqrt{lambda} |s-t|}}
                {2 mathrm{i} sqrt{lambda}} .
                end{eqnarray*}



                We want to compute the right hand side of the following equation:



                begin{eqnarray*}
                delta(s-t) = -frac{1}{2 pi mathrm{i}} int_{C_{infty}}
                G(s,t,lambda) d lambda
                end{eqnarray*}

                where the contour $C_{infty}$ contains all the spectrum (which is located in the imaginary
                line). As it is, this function as has a
                brunch cut at $0$, so we make the change of variables $lambda=mu^2$, $d lambda=
                2 mu d mu$
                , and unfold the integral where now $mu$ runs from $-infty$ to
                $infty$ along the imaginary axis. We have then



                begin{eqnarray*}
                -frac{1}{2 pi mathrm{i}} int_{C_{infty}}
                G(s,t,lambda) d lambda
                &=& -frac{1}{2 pi mathrm{i}} int_{-mathrm{i} infty}^{mathrm{i} infty}
                2 mu frac{mathrm{e}^{ mu |s -t|}}{2 mathrm{i}
                mu} d mu \
                &=&
                frac{1}{2 pi}
                int_{-mathrm{i} infty}^{mathrm{i} infty}
                mathrm{e}^{mu |s -t|} d mu.
                end{eqnarray*}



                We observe that that there is no need for the absolute bars in the exponent (check
                this by assuming $|s|>|t|$ and then $|t|>|s|$) so we write



                begin{eqnarray*}
                delta(s-t) = frac{1}{2 pi} int_{-mathrm{i} infty}^{mathrm{i} infty}
                mathrm{e}^{mu(s-t)} d mu.
                end{eqnarray*}



                At this moment we assume that $u(x)$ is a causal function, or simple
                a regular (no causal function) multiplied by the Heaviside function $H(x)$.
                Let us now multiply both sides of this equation by $u(x)$ and integrate
                over $s$ between $0$ and $infty$. Then



                begin{eqnarray*}
                u(t) = int_0^{infty} ds , u(s) frac{1}{2 pi} int_{-mathrm{i}infty}^
                {mathrm{i} infty}
                mathrm{e}^{mu(s - t)} d mu.
                = frac{1}{2 pi} int_{-mathrm{i} infty}^{mathrm{i} infty} d mu
                left ( int_0^{infty} ds , u(s) mathrm{e}^{mu s} right )
                ; mathrm{e}^{-mu t}
                end{eqnarray*}



                We define the expression inside parenthesis as $U(mu)$, so that we
                have the Laplace transform pair:



                begin{eqnarray*}
                U(mu) &=& int_0^{infty} ds , u(s) mathrm{e}^{mu s} \
                u(t) &=& frac{1}{2 pi} int_{-mathrm{i} infty}^{mathrm{i} infty} d mu
                U(t) mathrm{e}^{-t mu}.
                end{eqnarray*}







                share|cite|improve this answer














                share|cite|improve this answer



                share|cite|improve this answer








                edited Jan 25 at 20:05

























                answered May 24 '16 at 0:03









                Herman JaramilloHerman Jaramillo

                1,3391019




                1,3391019






























                    draft saved

                    draft discarded




















































                    Thanks for contributing an answer to Mathematics Stack Exchange!


                    • Please be sure to answer the question. Provide details and share your research!

                    But avoid



                    • Asking for help, clarification, or responding to other answers.

                    • Making statements based on opinion; back them up with references or personal experience.


                    Use MathJax to format equations. MathJax reference.


                    To learn more, see our tips on writing great answers.




                    draft saved


                    draft discarded














                    StackExchange.ready(
                    function () {
                    StackExchange.openid.initPostLogin('.new-post-login', 'https%3a%2f%2fmath.stackexchange.com%2fquestions%2f1796021%2fwhat-is-the-ode-that-gives-rise-to-the-laplace-transform%23new-answer', 'question_page');
                    }
                    );

                    Post as a guest















                    Required, but never shown





















































                    Required, but never shown














                    Required, but never shown












                    Required, but never shown







                    Required, but never shown

































                    Required, but never shown














                    Required, but never shown












                    Required, but never shown







                    Required, but never shown







                    Popular posts from this blog

                    Mario Kart Wii

                    What does “Dominus providebit” mean?

                    Antonio Litta Visconti Arese